Difference between revisions of "2019 AMC 12B Problems/Problem 2"

(Problem)
(Solution)
Line 4: Line 4:
 
==Solution==
 
==Solution==
  
answer should be 27
+
answer should be 27 (SuperWill)
  
 
==See Also==
 
==See Also==
 
{{AMC12 box|year=2019|ab=B|num-b=1|num-a=3}}
 
{{AMC12 box|year=2019|ab=B|num-b=1|num-a=3}}
 
{{MAA Notice}}
 
{{MAA Notice}}

Revision as of 12:52, 14 February 2019

Problem

If n is not a prime than (n-2) must be a prime. Which of the following choices provide a counter example for this statement?

Solution

answer should be 27 (SuperWill)

See Also

2019 AMC 12B (ProblemsAnswer KeyResources)
Preceded by
Problem 1
Followed by
Problem 3
1 2 3 4 5 6 7 8 9 10 11 12 13 14 15 16 17 18 19 20 21 22 23 24 25
All AMC 12 Problems and Solutions

The problems on this page are copyrighted by the Mathematical Association of America's American Mathematics Competitions. AMC logo.png